Quiz 2 Practice Problems


Problem 1. Compute each of the following volumes enclosed by two surfaces.

(a) The sphere \(x^2+y^2+z^2=a^2\) and cylinder \(x^2+y^2=ax\);

(b) The cone \(z=\sqrt{x^2+y^2}\) and the cylinder \(z^2=2x\);

(c) The two cylinders \(x^2+y^2=R^2\) and \(x^2+z^2=R^2\).


Problem 2. Find the center of mass of the following plane regions.

(a) \(D\) enclosed by \(y=\sqrt{2px}, \,\, x=x_0>0, \,\, y=0\);

(b) \(D\) is the semi-ellipse \(\left\{\left(x,y\right)\mid \frac{x^2}{a^2}+\frac{y^2}{b^2}\leq 1, y\geq 0 \right\}\);

(b) \(D\) enclosed by two disks \(\rho=a\cos\theta\) and \(\rho=b\cos\theta\) (\(0<a<b\), \(-\frac{\pi}{2}\leq \theta\leq \frac{\pi}{2})\).


Problem 3. Find the center of mass of the plane region \(D\) enclosed by \(y=x^2\) and \(y=x\), with density function

$$\mu \left(x,y\right) = x^2y.$$

Problem 4. Compute the center of mass of the following solid bodies (using triple integrals, of course). Assume the density is identically equal to \(1\).

(a) \(\Omega\) enclosed by \(z^2=x^2+y^2\) and \(z=1\);

(b) \(\Omega\) enclosed by \(z=\sqrt{A^2-x^2-y^2}\), \(z=\sqrt{a^2-x^2-y^2}\), and \(z=0\) (assuming \(A>a>0\));

(c) \(\Omega\) enclosed by \(z=x^2+y^2\), \(x+y=a\), \(x=0\), \(y=0\), \(z=0\).


Problem 5. Compute the moments of inertia \(I_x\), \(I_y\), \(I_0\) for the following plane regions.

(a) \(D=\left\{\left(x,y\right)\mid \frac{x^2}{a^2}+\frac{y^2}{b^2}\leq 1\right\}\);

(b) \(D\) enclosed by \(y^2=\frac{9}{2}x\) and \(x=2\);

(c) \(D\) is the rectangle \(\left[0,a\right]\times\left[0,b\right]\).


Problem 6. Consider the solid body \(\Omega\) (with constant density \(\rho_0\)) enclosed by the surface \(z=x^2+y^2\), \(z=0\), \(\left|x\right|=a, \left|y\right|=a\). Compute:

(1) The volume of \(\Omega\);

(2) The center of mass of \(\Omega\);

(3) The moment of inertia of \(\Omega\) with respect to the \(z\) axis.


Problem 7. Find the moment of inertial of a cylinder with respect to its axis of symmetry, assuming the height of the cylinder is \(h\), the radius of its base is \(a\), and the density is identically equal to \(1\).


Problem 8. Use triple integral to compute the volume of the unit ball \(\left\{\left(x,y,z\right)\mid x^2+y^2+z^2\leq 1\right\}\).


Answers

Problem 1.

(a) \(\frac{2}{3}a^3\left(\pi-\frac{4}{3}\right)\) \(\qquad\) (b) \(2\pi-\frac{32}{9}\) \(\qquad\) (c) \(\frac{16}{3}R^3\)

Problem 2.

(a) \(\bar{x} = \frac{3}{5}x_0\), \(\bar{y}=\frac{3}{8}\sqrt{2px_0}\). \(\quad\) (b) \(\bar{x}=0\), \(\bar{y}=\frac{4b}{3\pi}\). \(\quad\) (c) \(\bar{x}=\frac{a^2+ab+b^2}{2\left(a+b\right)}\), \(\bar{y}=0\).

Problem 3.

\(\bar{x} = \frac{35}{48}\), \(\bar{y} = \frac{35}{54}\).

Problem 4.

(a) \(\left(0,0,\frac{3}{4}\right)\) \(\quad\) (b) \(\left(0,0,\frac{3\left(A^4-^4\right)}{8\left(A^3-a^3\right)}\right)\) \(\quad\) (c) \(\left(\frac{2}{5}a, \frac{2}{5}a, \frac{7}{30}a^2\right)\)

Problem 5.

(a) \(I_x = \frac{1}{4}\pi ab^3\), \(I_y = \frac{1}{4}\pi a^3b\) \(\quad\) (b) \(I_x = \frac{72}{5}\), \(I_y=\frac{96}{7}\) \(\quad\) (c) \(I_x=\frac{1}{3}ab^3\), \(I_y=\frac{1}{3}a^3b\)

Problem 6.

(1) \(\frac{8}{3}a^4\) \(\quad\) (2) \(\bar{x}=\bar{y}=0,\,\, \bar{z}=\frac{7}{15}a^2\) \(\quad\) (3) \(\frac{112}{45}a^6\rho_0\)

Problem 7.

\(\frac{\pi}{2}a^4h\)

Problem 8.

\(\frac{4}{3}\pi\)


Solutions to Selected Problems

Problem 1 (a). The volume is bounded by two graphs \(z_\mathrm{top}=\sqrt{a^2-x^2-y^2}\) and \(z_\mathrm{bottom}=-\sqrt{a^2-x^2-y^2}\) over the region \(x^2+y^2=ax\). The volume is thus equal to the double integral

\begin{align*} &\iint_{\left\{\left(x,y\right)\mid x^2+y^2-ax\leq 0\right\}}\left[\sqrt{a^2-x^2-y^2}-\left(-\sqrt{a^2-x^2-y^2}\right)\right]dA\\ &=\iint_{\left\{\left(x,y\right)\mid x^2+y^2-ax\leq 0\right\}}2\sqrt{a^2-x^2-y^2}\,dA \end{align*}

Note that the region defined by \(x^2+y^2-ax\leq 0\) can be written in polar coordinates as \(r\leq a\cos\theta\), with \(\theta\) ranging from \(-\frac{\pi}{2}\) to \(\frac{\pi}{2}\). Thus the above integral equals

\begin{align*} \int_{-\frac{\pi}{2}}^{\frac{\pi}{2}}\int_0^{a\cos\theta}2\sqrt{a^2-r^2}\,rdr\,d\theta. \end{align*}

The inner integral can be computed via a substitution \(u=r^2\):

\begin{align*} &\int_0^{a\cos\theta}2\sqrt{a^2-r^2}\,rdr=\int_0^{a^2\cos^2\theta}\sqrt{a^2-u}\,du\\ &=-\frac{2}{3}\left(a^2-u\right)^{\frac{3}{2}}\Bigg|_{u=0}^{u=a^2\cos^2\theta}=-\frac{2}{3}a^3\sin^3\theta+\frac{2}{3}a^3=\frac{2}{3}a^3\left(1-\sin^3\theta\right). \end{align*}

Plug this back into the outer integral, one has

\begin{align*} \int_{-\frac{\pi}{2}}^{\frac{\pi}{2}}\frac{2}{3}a^3\left(1-\sin^3\theta\right)\,d\theta&=\frac{2}{3}a^3\int_{-\frac{\pi}{2}}^{\frac{\pi}{2}}\left(1-\sin^3\theta\right)\,d\theta\\ &=\frac{2}{3}a^3\left(\pi-\int_{-\frac{\pi}{2}}^{\frac{\pi}{2}}\sin^3\theta\,d\theta\right)=\frac{2}{3}a^3\left(\pi-\frac{4}{3}\right). \end{align*}

The computation of the last integral involving \(\sin^3\theta\) is standard (see page TA-3 in the textbook). It can be done as follows:

\begin{align*} \int_{-\frac{\pi}{2}}^{\frac{\pi}{2}}\sin^3\theta\,d\theta &= \int_{-\frac{\pi}{2}}^{\frac{\pi}{2}}\sin^2\theta\cdot\sin\theta\,d\theta\\ &=-\int_{-\frac{\pi}{2}}^{\frac{\pi}{2}}\left(1-\cos^2\theta\right)d\cos\theta=-2\int_{0}^{\frac{\pi}{2}}\left(1-\cos^2\theta\right)d\cos\theta\\ &\stackrel{t=\cos\theta}{=\!=\!=\!=\!=\!=}-2\int_1^0\left(1-t^2\right)dt=2\int_0^1\left(1-t^2\right)dt=2\left(1-\frac{2}{3}\right)=\frac{4}{3}. \end{align*}

Problem 1 (b). The volume is bounded by two graphs \(z_\mathrm{top}=2x\) and \(z_\mathrm{bottom}=\sqrt{x^2+y^2}\) over the region \(x^2+y^2-2x\leq 0\). (The boundary of the region, which is \(x^2+y^2-2x=0\), can be determined by computing the intersection of \(z=\sqrt{x^2+y^2}\) and \(z^2=2x\).) Similar to Problem 1 (a), this region can be represented in polar coordinates as \(r\leq 2\cos\theta\) with \(\theta\in\left[-\frac{\pi}{2},\frac{\pi}{2}\right]\). The volume can thus be computed as

\begin{align*} &\iint_{\left\{\left(x,y\right)\mid x^2+y^2-2x\leq 0\right\}}\left[2x-\sqrt{x^2+y^2}\right]dA=\int_{-\frac{\pi}{2}}^{\frac{\pi}{2}}\int_0^{2\cos\theta}\left(2r\cos\theta-r\right)\,rdr\,d\theta\\ =&\int_{-\frac{\pi}{2}}^{\frac{\pi}{2}}\left(2\cos\theta-1\right)\int_0^{2\cos\theta}r^2dr\,d\theta=\int_{-\frac{\pi}{2}}^{\frac{\pi}{2}}\left(2\cos\theta-1\right)\cdot\frac{8}{3}\cos^3\theta\,d\theta\\ =&\frac{16}{3}\int_{-\frac{\pi}{2}}^{\frac{\pi}{2}}\cos^4\theta\,d\theta-\frac{8}{3}\int_{-\frac{\pi}{2}}^{\frac{\pi}{2}}\cos^3\theta\,d\theta=\frac{16}{3}\times \frac{3\pi}{8}-\frac{8}{3}\times \frac{4}{3}=2\pi-\frac{32}{9}. \end{align*}

The integrals involving \(\cos^3\theta\) and \(\cos^4\theta\) can be found on page TA-3 in the textbook, or computed as follows:

\begin{align*} \int_{-\frac{\pi}{2}}^{\frac{\pi}{2}}\cos^3\theta\,d\theta&=\int_{-\frac{\pi}{2}}^{\frac{\pi}{2}}\cos^2\theta\,d\sin\theta\stackrel{u=\cos\theta}{=\!=\!=\!=\!=\!=}\int_{-1}^1\left(1-u^2\right)du=2-\frac{2}{3}=\frac{4}{3},\\ \int_{-\frac{\pi}{2}}^{\frac{\pi}{2}}\cos^4\theta\,d\theta&=\int_{-\frac{\pi}{2}}^{\frac{\pi}{2}}\cos^3\theta\,d\sin\theta=\sin\theta\cos^3\theta\Bigg|_{\theta=-\frac{\pi}{2}}^{\theta=\frac{\pi}{2}}-\int_{-\frac{\pi}{2}}^{\frac{\pi}{2}}\sin\theta\,d\cos^3\theta\\ &=0-\int_{-\frac{\pi}{2}}^{\frac{\pi}{2}}\sin\theta\cdot 3\cos^2\theta \left(-\sin\theta\right)\,d\theta\\ &=3\int_{-\frac{\pi}{2}}^{\frac{\pi}{2}}\sin^2\theta \cos^2\theta \,d\theta=\frac{3}{4}\int_{-\frac{\pi}{2}}^{\frac{\pi}{2}}\left(2\sin\theta\cos\theta\right)^2d\theta\\ &=\frac{3}{4}\int_{-\frac{\pi}{2}}^{\frac{\pi}{2}}\sin^2 2\theta\,d\theta=\frac{3}{4}\int_{-\frac{\pi}{2}}^{\frac{\pi}{2}}\frac{1-\cos 4\theta}{2}d\theta\\ &=\frac{3}{4}\left(\frac{\pi}{2}-\frac{1}{2}\int_{-\frac{\pi}{2}}^{\frac{\pi}{2}}\cos 4\theta\,d\theta\right)\\ &=\frac{3}{4}\left(\frac{\pi}{2}-\frac{1}{2}\times\frac{1}{4}\sin 4\theta\Bigg|_{\theta = -\frac{\pi}{2}}^{\theta=\frac{\pi}{2}}\right)=\frac{3}{4}\times\frac{\pi}{2}=\frac{3\pi}{8}. \end{align*}

Problem 1 (c). The volume is two graphs \(z_\mathrm{top}=\sqrt{R^2-x^2}\) and \(z_\mathrm{bottom}=-\sqrt{R^2-x^2}\) over the (disk of radius \(R\)) region \(x^2+y^2\leq R^2\). Thus the desired volume is

two_cylinders

\begin{align*} &\iint_{\left\{\left(x,y\right)\mid x^2+y^2\leq R^2\right\}}\left[\sqrt{R^2-x^2}-\left(-\sqrt{R^2-x^2}\right)\right]dA=\iint_{\left\{\left(x,y\right)\mid x^2+y^2\leq R^2\right\}}2\sqrt{R^2-x^2}\,dA\\ =&\int_0^{2\pi}\!\!\!\int_0^R 2\sqrt{R^2-r^2\cos^2\theta}\,\,rdr\,d\theta=\int_0^{2\pi}\!\!\!\int_0^R \sqrt{R^2-r^2\cos^2\theta}\,\,d\left(r^2\right)\,d\theta\\ =&4\int_0^{\frac{\pi}{2}}\!\!\!\int_0^R \sqrt{R^2-r^2\cos^2\theta}\,\,d\left(r^2\right)\,d\theta\\ &\!\!\!\!\stackrel{u=r^2}{=\!=\!=\!=\!=\!=}4\int_0^{\frac{\pi}{2}}\!\!\!\int_0^{R^2}\sqrt{R^2-u\cos^2\theta}\,\,du\,d\theta=4\int_0^{\frac{\pi}{2}}-\frac{2}{3\cos^2\theta}\left(R^2-u\cos^2\theta\right)^{\frac{3}{2}}\Bigg|_{u=0}^{u=R^2}\,d\theta\\ =&4\int_0^{\frac{\pi}{2}}\frac{2}{3\cos^2\theta}R^3-\frac{2}{3\cos^2\theta}\left(R^2-R^2\cos^2\theta\right)^{\frac{3}{2}}d\theta=4\int_0^{\frac{\pi}{2}}\frac{2R^3}{3\cos^2\theta}\left(1-\sin^3\theta\right)\,d\theta\\ =&\frac{8R^3}{3}\int_0^{\frac{\pi}{2}}\frac{1-\sin^3\theta}{\cos^2\theta}\,d\theta=\frac{16R^3}{3}. \end{align*}

Note that the last step follows from the following computation

\begin{align*} &\int_0^{\frac{\pi}{2}}\frac{1-\sin^3\theta}{\cos^2\theta}\,d\theta=\int_0^{\frac{\pi}{2}}\frac{\left(1-\sin\theta\right)\left(1+\sin\theta+\sin^2\theta\right)}{1-\sin^2\theta}\,d\theta\\ =&\int_0^{\frac{\pi}{2}}\frac{\left(1-\sin\theta\right)\left(1+\sin\theta+\sin^2\theta\right)}{\left(1-\sin\theta\right)\left(1+\sin\theta\right)}\,d\theta\\ =&\int_0^{\frac{\pi}{2}}\frac{1+\sin\theta+\sin^2\theta}{1+\sin\theta}\,d\theta=\int_0^{\frac{\pi}{2}}\frac{1+\sin\theta\left(1+\sin\theta\right)}{1+\sin\theta}\,d\theta\\ =&\int_0^{\frac{\pi}{2}}\frac{d\theta}{1+\sin\theta}+\int_0^{\frac{\pi}{2}}\sin\theta\,d\theta=\int_0^{\frac{\pi}{2}}\frac{d\theta}{1+\sin\theta}+1\\ =&\int_0^{\frac{\pi}{2}}\frac{d\theta}{1+2\sin\frac{\theta}{2}\cos\frac{\theta}{2}}+1\\ =&\int_0^{\frac{\pi}{2}}\frac{d\theta}{\sin^2\frac{\theta}{2}+\cos^2\frac{\theta}{2}+2\sin\frac{\theta}{2}\cos\frac{\theta}{2}}+1\\ =&\int_0^{\frac{\pi}{2}}\frac{d\theta}{\left(\sin\frac{\theta}{2}+\cos\frac{\theta}{2}\right)^2}+1\\ =&\int_0^{\frac{\pi}{2}}\frac{d\theta}{\cos^2\frac{\theta}{2}\left(\tan\frac{\theta}{2}+1\right)^2}+1\\ =&\int_0^{\frac{\pi}{2}}\frac{d\left(2\tan\frac{\theta}{2}\right)}{\left(\tan\frac{\theta}{2}+1\right)^2}+1\qquad\textrm{becase $\frac{d}{d\theta}\tan\frac{\theta}{2}=\frac{1}{2\cos^2\frac{\theta}{2}}$}\\ &\!\!\!\!\stackrel{u=tan\frac{\theta}{2}}{=\!=\!=\!=\!=\!=}\int_0^{1}\frac{2du}{\left(u+1\right)^2}\,d\theta+1=-\frac{2}{u+1}\Bigg|_{u=0}^{u=1}+1=-1+2+1=2. \end{align*}

The region enclosd by two cylinders of equal radii is a highly interesting object in geometry and goes a long way back in the pre-history of calculus. Check this article from Wolfram MathWorld and this page from TAMU for more details.

Problem 2 (a). By definition,

\begin{align*} m&=\iint_D\,dx\,dy=\int_0^{x_0}\!\!\!\int_0^{\sqrt{2px}}dy\,dx=\int_0^{x_0}\sqrt{2px}\,dx=\frac{2}{3}\sqrt{2px_0^3},\\ \bar{x}&=\frac{1}{m}\iint_Dx\,dx\,dy=\frac{1}{m}\int_0^{x_0}\!\!\!\int_0^{\sqrt{2px}}x\,dy\,dx=\frac{1}{m}\int_0^{x_0}\sqrt{2px^3}\,dx\\ &=\frac{3}{2\sqrt{2px_0^3}}\cdot\frac{2}{5}\sqrt{2px_0^5}=\frac{3}{5}x_0,\\ \bar{y}&=\frac{1}{m}\iint_Dy\,dx\,dy=\frac{1}{m}\int_0^{x_0}\!\!\!\int_0^{\sqrt{2px}}y\,dy\,dx=\frac{1}{m}\int_0^{x_0}\frac{1}{2}\cdot 2px\,dx\\ &=\frac{3}{2\sqrt{2px_0^3}}\cdot\frac{1}{2}px_0^2=\frac{3}{8}\sqrt{2px_0}. \end{align*}

Problem 3. By definition,

\begin{align*} m&=\iint_Dx^2y\,dA=\int_0^1\!\!\!\int_{x^2}^xx^2y\,dy\,dx=\int_0^1\frac{x^2}{2}\left(x^2-x^4\right)\,dx\\ &=\left(\frac{x^5}{10}-\frac{x^7}{14}\right)\Bigg|_{x=0}^{x=1}=\frac{1}{10}-\frac{1}{14}=\frac{1}{35},\\ \bar{x}&=\frac{1}{m}\iint_Dx\cdot x^2y\,dA=35\int_0^1\!\!\!\int_{x^2}^xx^3y\,dy\,dx=35\int_0^1\frac{x^3}{2}\left(x^2-x^4\right)\,dx\\ &=\frac{35}{2}\left(\frac{x^6}{6}-\frac{x^8}{8}\right)\Bigg|_{x=0}^{x=1}=\frac{35}{2}\cdot\frac{1}{24}=\frac{35}{48},\\ \bar{y}&=\frac{1}{m}\iint_Dy\cdot x^2y\,dA=35\int_0^1\!\!\!\int_{x^2}^xx^2y^2\,dy\,dx=35\int_0^1\frac{x^2}{3}\left(x^3-x^6\right)\,dx\\ &=\frac{35}{3}\left(\frac{x^6}{6}-\frac{x^9}{9}\right)\Bigg|_{x=0}^{x=1}=\frac{35}{3}\cdot\frac{3}{54}=\frac{35}{54}. \end{align*}

Problem 4 (a). By symmetry, \(\bar{x}=0\), \(\bar{y}=0\). The total mass is (note that the density is identically \(1\))

\begin{align*} m&=\iiint_\Omega dV=\int_0^1\iint_{\left\{\left(x,y\right)\mid x^2+y^2\leq z^2\right\}}\,dA\,dz\\ &=\int_0^1\pi z^2\,dz=\frac{\pi}{3}z^3\Bigg|_{z=0}^{z=1}=\frac{\pi}{3}, \end{align*}

thus the \(z\)-coordinate of the center of mass is

\begin{align*} \bar{z}&=\frac{1}{m}\iiint_\Omega z\,dV=\frac{3}{\pi}\int_0^1\iint_{\left\{\left(x,y\right)\mid x^2+y^2\leq z^2\right\}}z\,dA\,dz\\ &=\frac{3}{\pi}\int_0^1\pi z^2\cdot z\,dz=\frac{3}{\pi}\cdot\frac{\pi}{4}z^4\Bigg|_{z=0}^{z=1}=\frac{3}{4}. \end{align*}

Problem 5 (a). Direct computation gives

\begin{align*} I_y&=\iint_D x^2\,dA=\int_{-a}^a\!\!\!\int_{-\frac{b}{a}\sqrt{a^2-x^2}}^{\frac{b}{a}\sqrt{a^2-x^2}}x^2\,dy\,dx\\ &=\frac{2b}{a}\int_{-a}^ax^2\sqrt{a^2-x^2}\,dx=\frac{4b}{a}\int_{0}^ax^2\sqrt{a^2-x^2}\,dx\\ &\stackrel{x=\sin\theta}{=\!=\!=\!=\!=\!=}\frac{4b}{a}\int_{0}^{\frac{\pi}{2}}a^3\sin^2\theta\cos\theta\cdot a\cos\theta\,d\theta\\ &=4a^3b\int_{0}^{\frac{\pi}{2}}\left(\sin^2\theta\left(1-\sin^2\theta\right)\right)\,d\theta\\ &=4a^3b\left[\int_{0}^{\frac{\pi}{2}}\sin^2\theta\,d\theta-\int_{0}^{2\pi}\sin^4\theta\,d\theta\right]\\ &=4a^3b\left(\frac{\pi}{4}-\frac{3\pi}{8}\right)=\frac{1}{4}\pi a^3b. \end{align*}

(The integral involving \(\sin^4\theta\) is computed using the same idea as that used for integrating \(\cos^4\theta\) in the solution to Problem 1 (b).)

By symmetry, \(I_x=\frac{1}{4}\pi ab^3\). Thus

$$I_0=I_x+I_y=\frac{\pi}{4}ab\left(a^2+b^2\right).$$

Problem 6. Take a look at the graph of the function \(z=x^2+y^2\), as follows. (Rotate/Zoom with mouse.)

(1) The volume of \(\Omega\) equals to the volume under the graph \(z=x^2+y^2\) over the rectangle bounded by \(\left|x\right|\leq a\), \(\left|y\right|\leq a\). Thus

\begin{align*} \mathrm{Vol}\left(\Omega\right)&=\iint_{\left\{\left(x,y\right)\mid \left|x\right|\leq a, \left|y\right|\leq a\right\}}\left(x^2+y^2\right)\,dA\\ &=\int_{-a}^a\int_{-a}^a\left(x^2+y^2\right)\,dx\,dy\\ &=\int_{-a}^a \left(\frac{1}{3}x^3+xy^2\right)\Bigg|_{x=-a}^{x=a} \,dy\\ &=\int_{-a}^a\left(\frac{2}{3}a^3+2ay^2\right)\,dy\\ &=\frac{2}{3}a^3\cdot 2a+2a\cdot \frac{2}{3}a^3=\frac{8}{3}a^4. \end{align*}

(2) Since the density of \(\Omega\) is constant \(\rho_0\), the total mass is

$$m=\rho_0\cdot\mathrm{Vol}\left(\Omega\right)=\frac{8}{3}a^4\rho_0$$

The \(z\)-coordinate of the center of mass is thus

\begin{align*} \bar{z}&=\frac{1}{m}\iiint_{\Omega}\rho_0 z\,dV=\frac{1}{m}\int_{-a}^a\!\!\int_{-a}^a\!\!\int_0^{x^2+y^2}\rho_0 z\,dz\,dy\,dx\\ &=\frac{\rho_0}{m}\int_{-a}^a\!\!\int_{-a}^a\frac{1}{2}\left(x^2+y^2\right)^2\,dy\,dx\\ &=\frac{\rho_0}{2m}\int_{-a}^a\!\!\int_{-a}^a\left(x^4+2x^2y^2+y^4\right)\,dy\,dx\\ &=\frac{\rho_0}{2m}\int_{-a}^a\left(x^4y+\frac{2}{3}x^2y^3+\frac{1}{5}y^5\right)\Bigg|_{y=-a}^{y=a}dx\\ &=\frac{\rho_0}{2m}\int_{-a}^a\left(2ax^4+\frac{4a^3}{3}x^2+\frac{2a^5}{5}\right)\,dx\\ &=\frac{\rho_0}{2m}\left(2a\cdot\frac{1}{5}x^5+\frac{4a^3}{3}\cdot\frac{1}{3}x^3+\frac{2a^5}{5}x\right)\Bigg|_{x=-a}^{x=a}\\ &=\frac{3\rho_0}{16a^4\rho_0}\left(2a\cdot\frac{2}{5}a^5+\frac{4a^3}{3}\cdot\frac{2a^3}{3}+\frac{2a^5}{5}\cdot 2a\right)\\ &=\frac{3}{16 a^4}\cdot \frac{112}{45}a^6=\frac{7}{15}a^2. \end{align*}

By symmetry, \(\bar{x}=0\), \(\bar{y}=0\).

(3) The moment of inertia is

\begin{align*} I_0&=\iiint_\Omega \rho_0\left(x^2+y^2\right)dV=\rho_0\iiint_\Omega\left(x^2+y^2\right)dV\\ &=\rho_0\int_{-a}^a\!\!\int_{-a}^a\!\!\int_0^{x^2+y^2}\left(x^2+y^2\right)\,dz\,dy\,dx\\ &=\rho_0\int_{-a}^a\!\!\int_{-a}^a\left(x^2+y^2\right)^2\,dy\,dx\\ &=\rho_0\cdot \frac{112}{45}a^6=\frac{112}{45}a^6\rho_0. \end{align*}

Note that in the computation above we reused the intermediate result in (2) that

$$\int_{-a}^a\!\!\int_{-a}^a\left(x^2+y^2\right)^2\,dy\,dx=\frac{112}{45}a^6.$$

Problem 7. Consider a Cartesian coordinate system with origin at the geometric center of the cylinder, with \(z\)-axis aligned with the axis of symmetry. Then the cylinder occupies the following region in \(\mathbb{R}^3\):

$$\Omega = \left\{\left(x,y,z\right) \mid x^2+y^2\leq a^2, -\frac{h}2\leq z\leq \frac{h}{2}\right\}.$$

The desired moment of inertia is with respect to the \(z\)-axis. Therefore,

\begin{align*} I_0&=\iiint_{\Omega}\left(x^2+y^2\right)dV=\int_{-\frac{h}{2}}^{\frac{h}{2}}\iint_{\left\{\left(x,y\right)\mid x^2+y^2\leq a^2\right\}}\left(x^2+y^2\right)dA\,dz\\ &=\int_{-\frac{h}{2}}^{\frac{h}{2}}\int_0^{2\pi}\int_{0}^{a}r^2\cdot rdr\,d\theta\,dz=\int_{-\frac{h}{2}}^{\frac{h}{2}}\int_0^{2\pi}\frac{a^4}{4}\,d\theta\,dz\\ &=\int_{-\frac{h}{2}}^{\frac{h}{2}}2\pi\cdot \frac{a^4}{4}\,dz=\frac{\pi a^4}{2}h. \end{align*}

Problem 8. Note that the ball is symmetric so it doesn't matter which direction you pick to slice. Let's try slicing along the \(z\)-axis. The bounds in \(z\) is from \(-1\) to \(1\). For each fixed \(z\), the slice of the ball has equation \(x^2+y^2\leq 1-z^2\), i.e. it is a disk in the \(xy\)-plane of radius \(\sqrt{1-z^2}\). Thus

\begin{align*} \mathrm{Vol}\left(B\right)&=\iiint_BdV=\int_{-1}^1\iint_{\left\{\left(x,y\right)\mid x^2+y^2\leq 1-z^2\right\}}\,dA \,dz\\ &=\int_{-1}^1\mathrm{Area}\left(\left\{\left(x,y\right)\mid x^2+y^2\leq 1-z^2\right\}\right)\,dz\\ &=\int_{-1}^1\pi \left(1-z^2\right)\,dz=\pi\left(z-\frac{1}{3}z^3\right)\Bigg|_{z=-1}^{z=1}\\ &=\pi\left(\frac{2}{3}-\left(-\frac{2}{3}\right)\right)=\frac{4}{3}\pi. \end{align*}

useful links

social